LSAT and Law School Admissions Forum

Get expert LSAT preparation and law school admissions advice from PowerScore Test Preparation.

 Administrator
PowerScore Staff
  • PowerScore Staff
  • Posts: 8916
  • Joined: Feb 02, 2011
|
#100961
Complete Question Explanation

Strengthen. The correct answer choice is (D).

Answer choice (A):

Answer choice (B):

Answer choice (C):

Answer choice (D): This is the correct answer choice.

Answer choice (E):
 ChicaRosa
  • Posts: 111
  • Joined: Aug 23, 2016
|
#30600
Why is D correct and not E?

I thought D neither strengthened not weakened the stimulus since it mentioned that the study was determined by random basis while E addressed the conclusion of increasing one's life span.

Thanks!
 David Boyle
PowerScore Staff
  • PowerScore Staff
  • Posts: 836
  • Joined: Jun 07, 2013
|
#31269
ChicaRosa wrote:Why is D correct and not E?

I thought D neither strengthened not weakened the stimulus since it mentioned that the study was determined by random basis while E addressed the conclusion of increasing one's life span.

Thanks!

Hello ChicaRosa,

Answer E is not helpful; we really don't care why someone wants to exercise. Answer D is good because it helps preclude things like a reverse cause, e.g., it's not that the people who exercise become the healthiest (or tending to live the longest), it's that those who are already healthy then tend to exercise the most.

Hope this helps,
David
 cmorris32
  • Posts: 92
  • Joined: May 05, 2020
|
#79340
Hi PowerScore!

Based on my understanding of this stimulus, it is Cause and Effect Reasoning, and the conclusion is that exercise can cause one's life span to increase.

I know that one of the ways to strengthen a causal argument is to show that a data/statistical problem does not exist. Would answer choice D fall under this category? I was kind of thrown off by this answer because I feel like this is an uncommon way to strengthen a causal argument.

In addition, if a survey is presented in a stimulus, is it reasonable to believe that it is accurate?

Thank you in advance!
- Caroline
 jdconnect
  • Posts: 1
  • Joined: May 28, 2020
|
#79553
I am also seeking help with this question. I think Caroline is on the right track by acknowledging that D) strengthens in that it accounts for statistical bias. Keeping in mind that the conclusion is that exercise can increase one's lifespan, if the subjects are randomly selected to either exercise or refrain from exercise, then the possibility that there was some ulterior cause which led some subjects both to die and to refrain from exercise is eliminated.

But doesn't AC B) also eliminate a similar weakening idea? AC B) attributes the lack of exercise to a lack of time, thereby avoiding some ulterior cause which could also have also led to a decrease in lifespan.

I can't convince myself that B) does not strengthen the argument.
 Frank Peter
PowerScore Staff
  • PowerScore Staff
  • Posts: 99
  • Joined: May 14, 2020
|
#79655
Hi JD,

I think the problem with (B) is that it doesn't really have an impact on the argument. When we strengthen an argument we need to think about potential weaknesses with the argument and how we could eliminate those weaknesses. (D) eliminates the potential of a self-selection bias (i.e. the people who choose to exercise during the study may also engage in other healthy habits, making it difficult to say whether it's the exercise or some other habit causing an increased lifespan).

With (B), the reason why certain members of the study chose not to exercise doesn't really help us. Maybe if they found the time to exercise they would also have an increased lifespan, but we need to focus on tightening the connection between exercise and increased lifespan, and eliminating potential alternate causes. (D) does a better job at that because it eliminates one of the biggest weaknesses with the argument. And keep in mind, we are asked to choose the best answer choice. Even if we could think of some ways that (B) might help the argument out, (D) is a stronger choice.
User avatar
 ashpine17
  • Posts: 321
  • Joined: Apr 06, 2021
|
#94771
what is actually flawed abou the study? i'm assuming because we have to strengthen the argument it means it isn' tperfect. is it because we don't know whether the group was isolated from other potential factors that may affect their lifespan aside from exercise? the group tht doesn't exercise would be the control group right?
 Robert Carroll
PowerScore Staff
  • PowerScore Staff
  • Posts: 1787
  • Joined: Dec 06, 2013
|
#94811
ashpine,

We're told so little about these people that we have no idea what other factors may have been involved that might be relevant to their chances of dying during the study. The group that didn't exercise would I suppose be the control group, although, again, we don't even know if there is a "control" for any other factors because none of that is mentioned. The lack of mention doesn't mean it's not a problem - the lack of mention means we have no idea how it works, and thus the argument is extremely weak.

Robert Carroll
User avatar
 ashpine17
  • Posts: 321
  • Joined: Apr 06, 2021
|
#94842
this is a more general question but why are questions like these part of the assumption family? does that mean the answer choices strengthen the assumptions made in the stimulus or merely make them explicit?
User avatar
 katehos
PowerScore Staff
  • PowerScore Staff
  • Posts: 184
  • Joined: Mar 31, 2022
|
#94878
Hi ashpine17!

This question is a Strengthen question, which is part of the Help Family! In this family, you accept the truth of the answer choices themselves, then determine if they "help" strengthen the argument.

The correct answer is not necessarily an assumption of the argument, rather, what best supports the argument. This support can come from a multitude of different sources, for example, the correct answer may strengthen the argument by eliminate an alternate cause for the stated effect, showing that the data used was accurate, or even showing that when the cause does not occur, the effect does not occur.

In the context of this question, answer choice (D) strengthens the argument by eliminating problems with the data, since subjects were told to exercise during the study at random.

Hope this helps! :)
-Kate

Get the most out of your LSAT Prep Plus subscription.

Analyze and track your performance with our Testing and Analytics Package.